You are on page 1of 28

LEARNING ADVANCEMENT REVIEW CENTER

RM 413 DONA AMPARO BUILDING ESPANA BOULEVARD CORNER G. TOLENTINO ST.


SAMPALOC, MANILA PW 1
CONTACT # (02) 244 6342
6342 / 0915 537 1189 / 0943 595 5364

TAXATION GARCIA/ CO/ WONG

PART 1
BASIC PRINCIPLES -INHERENT POWERS OF TAXATION
1. Taxation is the act of laying a tax, i.e. the process or means by which the sovereign, through its lawmaking body, raises
income to defray the necessary expenses of government.

The power of taxation proceeds upon the theory that the existence of the government is a necessity, that it cannot
continue without means to pay its expenses and that for this means it has a right to compel all its citizens and property
within its limits to contribute.
A. True; True
B. True ; False
C. False ; True
D. False ; False

2. The principal purpose of taxation is to raise revenue for governmental needs (revenue purpose), which of the following
secondary purposes of taxation is not a compensatory purpose
A. To reduce excessive inequalities of wealth
B. To maintain high level of employment
C. To control inflation
D. To implement the police power of the State to promote the general welfare.

3. The basis is the reciprocal duties of protection and support between the State and its inhabitants. The State collects
taxes from the subjects of taxation in order that it may be able to perform the functions of government.

The citizens on the other hand, pays taxes in order that they may be secured in the enjoyment of the benefits of organized
society (benefits receivedtheory).

Taxes are the lifeblood of the Government and their prompt and certain availability are imperious (expecting obedience)
need.
A. True; True; True
B. True ; False; True
C. False ; True; True
D. False ; False; True

4. Which is not a manifestation of the lifeblood theory?


A. Imposition of tax even in the absence of Constitutional grant.
B. Right to select objects of taxation.
C. No injunction to enjoin (or stop) tax collection.
D. Fixing the amount or rate of the tax.

5. The power of taxation is comprehensive, plenary, unlimited and supreme. This power is, however, subject to inherent
and constitutional limitations.

It is subject to Constitutional and inherent limitations, hence, it is not an absolute power than can be exercised by the
legislature anyway it pleases.
A. True; True
B. True ; False
C. False ; True
D. False ; False

6. Which is not a characteristics or nature of the state’s power to tax?


A. It is inherent in sovereignty.
B. It is legislative in character.
C. It is subject to Constitutional and inherent limitations.
D. It is essentially administrative in character.

7. Which is not a basic principle of a sound tax system?


A. Fiscal Adequacy
B. Equality or Theoretical Justice
C. Administrative Feasibility
D. Eminent Domain

BASIC PRINCIPLES-LIMITATIONS ON POWER OF TAXATION

8. Which is not an inherent limitation of taxation?


A. International comity
B. Territorial jurisdiction
C. Double taxation
D. Due Process of Law

BASIC PRINCIPLES - TAX AND OTHER CHARGES

9. Which is not a characteristic of TAX?


A. It is an enforced contribution;
B. It is proportionate in character;
C. It is payable in money;

LEADTAX Pre Week Page 1 of 28


LEARNING ADVANCEMENT REVIEW CENTER LEAD
D. It is levied on persons and property within and without the jurisdiction of the State.

10. Which is false?


A. Tollis a sum of money collected for the use of something.
B. Special assessment is an enforced proportional contribution from owners of lands for special benefits resulting
from public improvements.
C. License fee or permit is a charge imposed under the police power for the purpose of regulations.
D. A Internal revenue is pecuniary aid directly granted by the government to an individual or private commercial
enterprise deemed beneficial to the public.

11. Shifting is the transfer of the burden of a tax by the original payer or the one to whom the tax was assessed or imposed to
another or someone else.
Tax evasion (tax dodging) is the use by the taxpayer of illegal means to defeat or lessen the payment of tax.
A. True; True
B. True ; False
C. False ; True
D. False ; False

POWERS AND AUTHORITY OF THE COMMISSIONER OF INTERNAL REVENUE

12. Which of the following is not a power of the commissioner of internal revenue?
A. Interpret tax laws and to decide tax cases.
B. Obtain information and to summon, examine, and take testimony of persons.
C. Make assessments and prescribe additional requirements for tax administration and enforcement.
D. Delegate the powers vested in him under the pertinent provisions of the Tax Code.

TAX REMEDIES- COMMISSIONER OF INTERNAL REVENUE

13. Which is not an authority of the commissioner of Internal Revenue?


A. Compromise the payment of any internal revenue tax;
B. Cancel or abate tax liability;
C. Credit or refund.
D. Enact tax laws.

TAX REMEDIES- GOVERNMENT

14. Which is a civil remedy for the collection of internal revenue taxes, fees, or charges, and any increment thereto resulting
from delinquency?
a) Distraint of personal property and
b) Levy upon real property and interest in or rights to real property.
c) Civil action or
d) Criminal action.
A. a and b
B. b and c
C. a,b and c
D. a, b, c, and d

TAX REMEDIES- GOVERNMENT

15. (Phil. CPA) A taxpayer filed her income tax return for 2000 on April 15, 2001 and paid the tax of P12,000. Upon audit of the
BIR, an assessment notice was issued on May 2, 2003 requiring the taxpayer to pay a deficiency tax of P50,000 not later
than July 15, 2003. The taxpayer may:
A. Go to the Supreme Court if the issues involved are pure question of law;
B. Ignore the assessment as the date of collection is beyond three (3) years as the taxable year covers 2000;
C. Request for an extension of time to pay the deficiency income tax;
D. Go to the Court of Tax Appeals to appeal the assessment made by the BIR.

16. (Phil. CPA modified) The following are related to the tax assessment of a taxpayer:
Date when assessment was received October 1, 2017
Petition for reconsideration was filed with BIR October 9, 2017
BIR decision of denial was received July 2, 2018
Second request for reconsideration was filed with the BIR July 15, 2018
Date revised assessment was received July 5, 2019

The last day to appeal to the Court of Tax Appeals is on:


A. July 15, 2019;
B. August 6, 2019;
C. August 4, 2019;
D. September 5, 2019.

Additions to Tax
17. (Phil. CPA Modified) Pio Company did not file his monthly VAT declaration for the month of January, 2017, which was due
for filing on February 20, 2017.

He was notified by the BIR of his failure to file the declaration, for which reason, he filed his declaration and paid the tax
only after the said notice on June 30, 2018. The tax due per monthly VAT declaration was P100,000.

LEAD TAX Pre Week Page 2 of 28


LEARNING ADVANCEMENT REVIEW CENTER LEAD
How much was the total amount due on June 30, 2018?
A. P100,000.00
B. P172,222.23
C. P177,214.62
D. P277,214.62

18. (Adapted) XYZ Corporation filed its income tax return for calendar year 1997 with a net taxable income of P500,000. At
the applicable income tax rate of 35% for the year 1997, its income tax amounted to P175,000. However, upon
investigation, it was disclosed that its income tax return was false or fraudulent because it did not report a taxable income
amounting to another P500,000. On its net income of P1,000,000, per investigation, the income tax due is P350,000.
Deducting its payment per return filed, the deficiency, excluding penalties, amounted to P175,000. It was duly informed of
this finding through the Preliminary Assessment Notice. Failing to protest on time against the preliminary assessment
notice, a formal letter of demand and assessment notice was issued on May 31, 1999 calling for payment of the deficiency
income tax on or before June 30, 1999.
a. How much was the total amount due?
b. Assuming that the calendar year 1997 deficiency income tax assessment against XYZ Corporation was not paid by
June 30, 1999, the deadline for payment of the assessment and assuming further that this assessment had already
become final and collectible. The corporation paid its tax assessment only by July 31, 1999.
a. b.
A. P175,000.00 P175,000.00
B. P304,771.68 307,784.25
C. P304,771.68 P312,863.79
D. P42, 271.68 P 45,284,25

19. (Phil. CPA) For filing false and fraudulent return, a surcharge is imposed. Which of the following is correct?
A. 50% administrative penalty
B. 50% criminal penalty
C. 25% plus 50%
D. 25% criminal penalty

ELECTRONIC FILING OF TAX RETURNS AND PAYMENT OF TAXES


20. It refers to the control number issued by the AAB to the BIR to confirm that tax payment has been credited to the account
of the government or recognized as revenue (internal revenue tax collection) by the Bureau of Treasury.
A. Electronic Filing and Payment System (EFPS)
B. Filing Reference Number
C. Confirmation Number
D. Acknowledgement Number

Receipts, Invoices and Books of Accounts

21. In cases where the sales or receipts in the preceding year exceed P30,000, receipt or invoice is issued for each sale or
transfer of merchandise or for services rendered valued at:
A. Not less than P10;
B. Not less than P20;
C. Not less than P25;
D. Regardless of the value.

22. When the quarterly sales, earnings, receipts or output exceed P50,000, what books of account shall be maintained by a
business entity?
A. Simplified set of bookkeeping records;
B. Journal and a ledger or their equivalents;
C. Either simplified set of bookkeeping records or journal and a ledger or their equivalents;
D. No books of account required.

TRANSFER TAXES
23. (ADAPTED) Which of the following is not included in the gross estate of a resident decedent who is under conjugal
partnership of gains?
A. House and lot inherited by the decedent during the marriage from his who predeceased the decedent.
B. Condominium unit transferred to his son for an insufficient consideration.
C. Jewelry purchased by the decedent during the marriage with his exclusive money of the decedent.
D. SMC Shares of stock inherited by the decedent’s surviving spouse during the marriage

24. Which of the following is not included in the gross estate of a resident decedent who is under conjugal partnership of gains?
A. House and lot inherited by the decedent during the marriage from his who predeceased the decedent.
B. Condominium unit transferred to his son for an insufficient consideration.
C. Jewelry purchased by the decedent during the marriage with his exclusive money of the decedent.
D. SMC Shares of stock inherited by the decedent’s surviving spouse during the marriage

25and 26. are based on the following:

25. (Adapted) A decedent dies on January 15, 2013. The interment is on January 31, 2013. The gross estate of the decedent is
P4,000,000. The following expenses are presented to you:
Expenses during the wake borne by relatives and friends P30,000
Mourning clothing of the widow and unmarried minor children 20,000
Obituary notice in a newspaper 50,000
Card of thanks published in a newspaper 30,000
Burial plot 80,000
Flowers placed on top of the burial lot when relatives visited on February 14, 2013 5,000

LEAD TAX Pre Week Page 3 of 28


LEARNING ADVANCEMENT REVIEW CENTER LEAD
Expenses during a family’s gathering to commemorate the 40thday after death of the 15,000
decedent on February 23, 2013
Accountant’s fee in gathering to commemorate the 40th day after death of the decedent 50,000
on February 23, 2013
Lawyer’s fee in representing the estate in case filed on August 15, 2013 70,000
Executor’s fee
January 15, 2013 to July 15, 2013 P120,000
July 16, 2013 to December 31, 2013 120,000 240,000

The deductible funeral expenses are:


A. P150,000.
B. P180,000.
C. P200,000.
D. none of the choices.

26. The deductible judicial expenses are:


A. P360,000.
B. P240,000.
C. P170,000.
D. none of the choices.

27.(Adapted) The following taxes are claimed as deduction from the gross estate of a decedent who dies on July 1, 2013.
Income tax on income earned from January 1 to June 30, 2013 P100,000
Income tax on income earned from July 1, 2013 to December 31, 2013 120,000
Real estate tax paid, January 1 to June 30, 2013 50,000
Unpaid real estate tax, July 1 to December 31, 2013 50,000
Unpaid property tax, January 1 to June 30, 2013 60,000

The correct amount of deductible taxes shall be:


A. P380,000.
B. P210,000.
C. P170,000.
D. none of the choices.

28.Which of the following shall be allowed as deduction from the gross estate of a non-resident alien decedent?

Old Law Train Law


A. Family home deduction Standard deduction / Transfer for public use
B. Medical expenses Standard deduction / Transfer for public use
C. Standard deduction Standard deduction / Transfer for public use
D. Transfer for public use Standard deduction / Transfer for public use

29.Which is of the following period for filing is true?


Old Law Train Law
Notice of death is filed 2 months after the decedent’s death Not required
Estate tax returns are filed 6 months after the decedent’s death 1 year after the decedent’s death

Notice of death is filed Estate tax returns are filed


A. True True
B. True False
C. False False
D. False True

30.S1-Under the TRAIN Law, the standard deduction is now P5,000,000


S2-Under the TRAIN Law, a CPA certification is required if the gross estate exceeds P5,000,000.
S3-Under the TRAIN Law, payment of estate tax is now allowed within 2 years from the date of death without civil
penalties or interest.

S1 S2 S3
A. True True True
B. True True False
C. True False False
D. False False False

31. to 34. are based on the following:


Harry had the following donations/transactions:
Date Donations
January 1 Donated a P150,000 diamond ring to her sister who
was getting married in March 15 of the same year.
March 1 Sold her personal car valued at P500,000 for
P200,000 to his uncle.
April 1 Sold his residential house to his brother for P1,500,000. The fair market value of the residential
house at the time of sale was P2,000,000.
June 1 Donated P100,000 to BinanCityfor public purpose and P50,000 to her brother who graduated from De La
Salle University.

31.How much was the donor’s tax for the gift made on January 1?
Old Law Train Law
A. P 30,000 P0

LEAD TAX Pre Week Page 4 of 28


LEARNING ADVANCEMENT REVIEW CENTER LEAD
B. P 1,000 P0
C. P 800 P 30,000
D. P 30,000 None of the choices

32.How much was the donor’s tax for the gift made on June 1?
Old Law Train Law
A. P 32,000 P 9,000
B. P 14,000 P 30,000
C. P 2,000 P 3,000
D. None of the choices None of the choices

33. Donation is completed:


A. when the donee accepts the donation and the acceptance is made known to the donor.
B. when the property donated is delivered actually or constructively to the donee.
C. when the donor’s tax is finally paid.
D. when it is determined that the donor has the capacity to make donations.

34.Donor’s return shall be filed within:


A. thirty (30) days after the donor executes a Deed of Donation.
B. thirty (30) days after the date the gift is made.
C. six (6) months after the date the gift is made
D. thirty (30) days after the donor pays the donor’s tax.

BUSINESS TAXES (Theories)

35.First statement: Only VAT-registered taxpayers are required to pay Value-Added Tax.

Second statement: Taxpayers whose gross annual sales or receipts exceed the VAT threshold amount are required to pay
Value-Added Tax in all cases.

A. Both statements are correct


B. Both statements are incorrect
C. Only the first statement is correct
D. Only the second statement is correct.

For Items 36 and 37

36.A VAT-registered lessor of apartment houses has the following gross rentals for the first quarter of the current year:
Gross rentals of 30 units at P10,000 per month per unit P3,600,000
Gross rentals of 10 units at P15,000 per month per unit 1,800,000
Total .................................................. P5,400,000

The VAT-registered lessor of apartment houses shall be:


A. subject to VAT on his gross rentals of P5,400,000.
B. subject to VAT on his gross rentals of P1,800,000.
C. subject to VAT on his gross rentals of P3,600,000.
D. exempt from VAT on his total gross rentals.

37.Under the TRAIN Law, the VAT-registered lessor of apartment houses shall be:
A. subject to VAT on his gross rentals of P5,400,000.
B. subject to VAT on his gross rentals of P1,800,000.
C. subject to VAT on his gross rentals of P3,600,000.
D. exempt from VAT on his total gross rentals.

38. The following transactions during the month of January, 2013 are recorded in the books of a VAT-registered taxpayer:
Purchase of three (3) office equipment at P150,000 each
Purchase of five (5) office furniture at P50,000 each
Importation of vehicle for transport used in business, P1,500,000

Based on the above data:


A. The total input taxes on the above transactions shall be amortized.
B. The input tax on office equipment and office furniture shall be credited in full.
C. Only the VAT on importation of vehicle for land transport shall be amortized.
D. The total input taxes on the above transactions may be amortized or credited in full depending on the choice of the
taxpayer.

39. Which of the following is allowed presumptive input tax?


A. Processor of sardines, mackerel and milk
B. Manufacturer of raw sugar
C. Processor of canned fruits
D. Manufacturer of packed instant champorado

40. The quarterly VAT return is filed:


A. Within twenty (20) days following the end of the quarter.
B. Within twenty-five (25) days following the end of the quarter.
C. Within sixty (60) days following the end of the quarter.
D. When the VAT payable is finally determined.

LEAD TAX Pre Week Page 5 of 28


LEARNING ADVANCEMENT REVIEW CENTER LEAD

41. One of the following is subject to the 3% percentage tax under Sec. 116:
A. Seller of fresh mangoes whose annual gross sales do not exceed the VAT threshold amount
B. Seller of canned fish whose annual gross sales exceed the VAT threshold amount
C. Seller of refined sugar whose annual gross sales do not exceed the VAT threshold amount
D. Seller of office supplies who is VAT-registered but whose gross annual sales do not exceed the VAT threshold
amount.

42. Which of the following is subject to common carriers tax?


A. Transportation contractors by land on their transport of goods and cargoes
B. Person who transports passengers by air
C. Domestic carrier by land relative their transport of passengers.
D. Person who transports goods and cargoes by sea

43. (Easy) Which of the following is not subject to franchise tax?


A. Franchise grantee of radio and/or television broadcasting whose gross annual receipts of the preceding year does
not exceed P10,000,000
B. Franchise grantees of gas and water utilities
C. PAGCOR and its licensees and franchisees
D. Franchise grantee of electric utilities

44. Rentals of property, real or personal, received by bank and non-bank financial intermediaries performing quasi-banking
functions are:
A. subject to Value-added Tax at 12%.
B. subject to Gross Receipts Tax at 7%.
C. subject to Gross Receipts Tax if the annual gross rentals do not exceed the VAT threshold amount.
D. not subject to any tax.

45.Which of the following is not subject to amusement tax?


A. Proprietor of videoke bars
B. Operator of cockpits
C. Lessee of bowling alleys
D. Lessee jai-alai and racetracks

For Items 46 and 47

46. Mr. X, a minimum wage earner, works for 123 Co. Mr. X is not engaged in business nor has any other source of income
other than his employment. For 2018, Mr. X earned a total compensation income of P135,000.00.The taxpayer contributed
to the SSS, Philhealth, and HDMF amounting to P5,000.00 and has received 13th month pay of P11,000.00. How much is
his income tax due?
A. EXEMPT
B. P135,000.00
C. P119,000.00
D. None of the choices

47. The following year, Mr. X earned, aside from his basic wage, additional pay of P140,000.00 which consists of the overtime
pay (P80,000.00), night shift differential (P30,000.00), hazard pay (P15.000.00) and holiday pay (P15,000.00). He has the
same benefits and contributions as in item 46. How much is his taxable income?
A. EXEMPT
B. P135,000.00
C. P140,000.00
D. P259,000.00

48. Ms. Terry operates a convenience store while she offers bookkeeping services to her clients. In 2018, her gross sales
amounted to P800,000.00, in addition to her receipts from bookkeeping services of P300,000.00. She already signified her
intention to be taxed at 8% income tax rate in her 1st quarter return. How much is her income tax due?
A. EXEMPT
B. P1,100,000.00
C. P 850,000.00
D. P 68,000.00

49. Ms. Terry above, failed to signify her intention to be taxed at 8% income tax rate on gross sales in her initial Quarterly
Income Tax Return, and she incurred cost of sales and operating expenses amounting to P600,000.00 and P200,000.00,
respectively, or a total of P800,000.00, how much is her income tax due?
A. EXEMPT
B. P1,100,000.00
C. P 300,000.00
D. P 10,000.00

50. Mr. Yoso signified his intention to be taxed at 8% income tax rate on gross sales in his 1st Quarter Income Tax Return. He
has no other source of income, His total sales for the first three (3) quarters amounted to P3,000,000.00 with 4th quarter
sales of P3,500,000.00.

How much is his annual income tax payable?


A. EXEMPT
B. P1,580,000.00
C. P509,200.00
D. P289,200.00

LEAD TAX Pre Week Page 6 of 28


LEARNING ADVANCEMENT REVIEW CENTER LEAD
51. Ms. RSVP is a prominent independent contractor who offers architectural and engineering services. Since her career
flourished, her total gross receipts amounted to P4,250,000.00 for taxable year 2018. Her recorded cost of service and
operating expenses were P2,150,000.00 and P1,000,000.00, respectively. How much is her tax due?
A. EXEMPT
B. P4,250,000.00
C. P1,100,000.00
D. P220,000.00

52. In 2018, Mr. Swabe owns a nightclub and videoke bar, with gross sales/receipts of P2,500,000.00. His cost of sales and
operating expenses are P1,000,000.00 and P600,000.00, respectively, and with non-operating income of P100,000.00. How
much is his tax due under graduated rates? Can he opt to be taxed at 8%?
Graduated Rates 8%
A. P1,000,000.00 Yes
B. P1,000,000.00 No
C. P 190,000.00 Yes
D. P 190,000.00 No

53. Mr. Madz, a Financial comptroller of JAC Company, earned annual compensation in 2018 of P1,500,000.00, inclusive of 13th
month and other benefits in the amount of P120,000.00 but net of mandatory contributions to SSS and Philhealth. Aside
from employment income, he owns a convenience store, with gross sales of P2,400,000. His cost of sales and operating
expenses are P1,000,000.00 and P600,000.00, respectively, and with non-operating income of P100,000.00. How much is
his tax due if
A. He opted an Eight Percent (8%) income tax rate on Gross Sales
B. He did not Opt for 8% income tax on Gross Sales/Receipts and other non-operating income

Graduated Rates 8%
A. P313,000.00 P1,410,000.00
B. P200,000.00 P2,310,000.00
C. P 513,000.00 P589,200.00
D. None of the choices None of the choices

54. On February 7, 2019, taxpayer tendered his resignation to concentrate on his business. His total compensation income
amounted to P150,000.00, inclusive of benefits of P20,000.00. His business operations for the taxable year 2019 remains
the same. How much is his tax due and payable if he opted for the eight percent (8%) income tax rate?
A. EXEMPT
B. P80,000.00
C. P130,000.00
D. P200,000.00

55. Mr. Wayne, an officer of BATS International Corp., earned in 2018 an annual compensation of P1,200,000.00, inclusive of
the 13th month and other benefits in the amount of P120,000.00. Aside from employment income, he owns a farm, with
gross sales of P3,500,000. His cost of sales and operating expenses are P1,000,000.00 and P600,000.00, respectively, and
with non-operating income of P100.000.00. How much is his tax due for 2018?
A. EXEMPT
B. P80,000.00
C. P130,000.00
D. P200,000.00

56. A lessor rents his 15 residential units for P14,500 per month. During the taxable year, his accumulated gross receipts
amounted to P2,610,000. Which is false?
A. He is not subject to VAT since the monthly rent per unit does not exceed P15,000.
B. He is also not subject to 3% Percentage Tax.
C. He is not subject to VAT whether he is a VAT registered entity or not.
D. He is subject to VAT whether he is a VAT registered entity or not.

57. Using the same data in number 56, assuming he has 20 residential units with the same monthly rent per unit and his
accumulated gross receipts during the taxable year amounted to P3,480,000, which of the following statement is false?
A. He is not subject to VAT since the monthly rent per unit does not exceed P15,000.
B. He is also not subject to 3% Percentage Tax.
C. He is not subject to VAT whether he is a VAT registered entity or not.
D. He is subject to VAT whether he is a VAT registered entity or not.

58. A lessor rents his 15 residential units for P15,500 per month. During the taxable year, his accumulated gross receipts
amounted to P2,790,000, which of the following statements is true?
A. He is not subject to VAT since the monthly rent per unit does not exceed P15,000.
B. He is also not subject to 3% Percentage Tax.
C. He is not subject to VAT whether he is a VAT registered entity or not.
D. He is not subject to VAT since his accumulated gross receipts did not exceed P3,000,000. However he is subject to
a 3% percentage tax.

59. Using the same data in number 58, assuming he has 20 residential units with the same monthly rent per unit and his
accumulated gross receipts during the taxable year amounted to P3,720,000, which of the following statements is true?
A. He is subject to VAT since the monthly rent per unit exceeds P15,000 and the accumulated earnings exceeded
P3,000,000 .
B. He is subject to 3% Percentage Tax.
C. He is not subject to VAT whether he is a VAT registered entity or not.
D. He is not subject to VAT since his accumulated gross receipts did not exceed P3,000,000. However he is subject to
a 3% percentage tax.

LEAD TAX Pre Week Page 7 of 28


LEARNING ADVANCEMENT REVIEW CENTER LEAD
60. A lessor rents his 2 commercial and 10 residential units for monthly rent of P60,000 and P15,000 per unit, respectively.
During the taxable year, his accumulated gross receipts amounted to P3,240,000 (P1,440,000 from commercial units and
P1,800,000 from residential units). Which of the following statements are false?
A. The P1,440,000 from commercial units is not subject to VAT since it did not exceed P3,000,000.
B. The P1,440,000 from commercial units is, however, subject to 3% Percentage Tax.
C. The P1,800,000 accumulated receipts from the residential units are not subject to Percentage Tax and exempt from
VAT since the monthly rent is not more than P15,000.
D. None of the above

61. Mr. JMLH signified his intention to be taxed at “8% income tax in lieu of the graduated income tax rates and percentage tax
under Section 116” in his 1st Quarter Income Tax. However, his gross sales/receipts during the taxable year have exceeded
the VAT threshold as follows:
January P250,000
February 250,000
March 250,000
April 250,000
May 250,000
June 250,000
July 250,000
August 250,000
September 250,000
October 1,000,000
November 1,000,000
December 1,000,000
Total gross sales/receipts P5,250,000
Which statement is false?
A. Mr. JMLH lost the option to pay the 8% commuted tax rate when his gross sales/receipts exceeded the three million
threshold during the 4th Quarter.
B. For business tax purposes, he is subject to the 12% VAT prospectively starting November 2018.
C. He is also required to update his registration from non-VAT to VAT on or before November 30, 2018.
D. None of the above

62. WPM is a rice dealer. His total annual gross sales and/or receipts do not exceed Three Million (P3,000,000.00), allowing him
to avail the following:
(a) WPM is a VAT-exempt taxpayer. He may elect to avail of the optional registration for VAT of exempt person under
Section 236 (H) of the 1997 Tax Code, as amended. Upon election of such option, he shall not be entitled to cancel his VAT
registration for the next three (3) years;
(b) WPM may elect to pay the 8% commuted tax rate on gross sales or receipts and other non-operating income in lieu of
the graduated income tax rates and the percentage tax under Section 24(A)(2)(b) of the 1997 Tax Code, as amended, since
his gross sales or receipts did not exceed Three Million Pesos (P3,000,000) during the taxable year. If he elects to pay the
8% commuted tax, he shall not be allowed to avail of the optional registration for VAT of exempt person provided by
Section 236(H) of the 1997 Tax Code, as amended.
A. True ; True
B. True ; False
C. False; True
D. False; False

63. What are the transactions which are no longer subject to zero-percent (0%)?
1. Sale of gold to BSP
2. Foreign-currency denominated sales
A. Both 1 and 2
B. 1 only
C. 2 only
D. Neither 1 nor 2

64. What are the transactions that will now be subject to twelve percent (12%) and no longer be subject to zero percent (0%),
upon the successful establishment and implementation of an enhanced VAT refund system by the Department of Finance
(DOF)?
1. The sale of raw materials or packaging materials to a non-resident buyer for delivery to a resident local export-oriented
enterprise to be used in manufacturing, processing, packing or repacking in the Philippines of the said buyer's goods, paid
for in acceptable foreign currency, and accounted for in accordance with the rules and regulations of the BSP;
2. The sale of raw materials or packaging materials to an export-oriented enterprise whose export sales exceed seventy
percent (70%) of total annual production;
3. Transactions considered export sales under Executive Order No. 226, otherwise known as the Omnibus Investments
Code of 1987, and other special laws
4. Processing, manufacturing or repacking goods for other persons doing business outside the Philippines which goods are
subsequently exported where the services are paid for in acceptable foreign currency and accounted for in accordance
with the rules and regulations of the BangkoSentral ng Pilipinas (BSP); and
5. Services performed by subcontractors and/or contractors in processing, converting, or manufacturing goods for an
enterprise whose export sales exceeds seventy percent (70%) of total annual production.
A. 1,2,3,4 and 5
B. 1,2,3 and 4
C. 1,2 and 3
D. 1 and 2

65. 1. Sale of real properties utilized for specialized housing as defined under RA No. 7279, and other related laws, such as RA
No. 7835 and RA No. 8763, wherein price ceiling per unit is Php 450,000.00 or as may from time to time be determined by
the HUDCC and the NEDA and other related laws;
2. Sale of residential lot valued at One Million Five Hundred Thousand Pesos (P1,500,000.00) and below, or house and lot
and other residential dwellings valued at Two Million Five Hundred Thousand Pesos (P2,500,000.00) and below, as adjusted

LEAD TAX Pre Week Page 8 of 28


LEARNING ADVANCEMENT REVIEW CENTER LEAD
using latest Consumer Price Index values. (If two or more adjacent lots are sold or disposed in favor of one buyer, for the
purpose of utilizing the lots as one residential lot, the sale shall be exempt from VAT only if the aggregate value of the lots
do not exceed One Million Five Hundred Thousand Pesos (P1,500,000.00). Adjacent residential lots, although covered by
separate titles and/or separate tax declarations, when sold or disposed to one and the same buyer, whether covered by one
or separate Deed of Conveyance, shall be presumed as a sale of one residential lot.)
Under the TRAIN Law which is true?
A. Both 1 and 2
B. 1 only
C. 2 only
D. Neither 1 nor 2

66. (Adapted) A VAT-registered public works contractor has the following data on services rendered in the Philippines for the
first quarter of 2007 (VAT-exclusive):
Contract price (foreign clients doing business
outside the Philippines) ($1:P50) $ 100,000
Contract price (private sector clients) P 5,000,000
Contract price (Government) 3,000,000
Collections from foreign clients $ 100,000
Collections from Government contracts 1,000,000
Collections from private sector clients 2,000,000
Purchases during the quarter
(used in private sector clients contracts) 800,000
Payments for services of a VAT-registered sub-
contractor (used in Government contracts) 400,000
Purchases during the quarter (used in
private sector clients and
government contracts only) 300,000
How much is the VAT payable for the quarter using 12% VAT?
A. P190,000
B. P170,000
C. P120,000
D. None

ITEMS 67 AND 68 Vanderwoodsen is a radio-TV broadcasting franchise grantee. The previous year, its gross receipts did not
exceed P 10,000,000. In the first month of the current year, it had the following data:
Gross receipts, sale of airtime P 2,000,000
Payments received from user of radio station’s
communications facilities for overseas
communications 500,000
Business expenses 700,000

67. How much was the franchise tax due?


A. P75,000
B. P60,000
C. P50,000
D. P40,000

68. How much was the overseas communications tax?


A. P250,000
B. P200,000
C. P75,000
D. P50,000

(Adapted)69 to 73 are based on the following: Audrey Foam Manufacturing Corp. is a VAT-registered enterprise. It has the
following data taken from its books of accounts for the first quarter of the fiscal year starting May 1, 2014 and ending April
30, 2015):
Domestic sale of goods P5,000,000
Sale of goods to senior citizens, net of 20% discount 800,000
Beginning inventory 300,000
Domestic purchases of goods for use in all transactions 500,000
Importation of goods for use in all transactions 700,000
Ending inventory 200,000
Payment for services for all transactions 450,000
Purchase of vehicle for land transport for use in all transactions (estimated life 5 years purchased
May 1, 2014) 2,500,000
Maintenance expense for the vehicle for land transport 100,000
Other operating expenses (including salaries of P100,000 given to senior citizen-employees)
500,000
Assistance given under Adopt-A-School Program to a state college 100,000
Monthly VAT paid (May and June) 300,000

69. How much is the creditable input tax?


A. P250,000
B. P198,000
C. P165,000
D. None of the choices

70. How much is the VAT payable?


A. P435,000

LEAD TAX Pre Week Page 9 of 28


LEARNING ADVANCEMENT REVIEW CENTER LEAD
B. P222,000
C. P135,000
D. None of the choices

71. How much is the total deductible expenses for income tax purposes?
A. P1,324,000
B. P1,124,000
C. P1,024,000
D. None of the choices

72. How much is the taxable net income for the quarter?
A. P3,552,000
B. P3,352,000
C. P2,352,000
D. None of the choices

73. When shall the income tax return and the quarterly VAT return be filed for the first fiscal quarter?
Quarterly ITR Quarterly VAT Return
A. September 30, 2014 September 30, 2014
B. September 25, 2014 August 31, 2014
C. November 15, 2014 August 20, 2014
D. September 29, 2014 August 25, 2014

74. Under the TRAIN Law,the following passive income are subject to
Resident/ Citizen NRA-NETB
A. PCSO winning amounting to P11,000 = 10% FWT PCSO winning amounting to P11,000= 10% FWT
B. PCSO winning amounting to P11,000 = 20% FWT PCSO winning amounting to P11,000= 25% FWT
C. PCSO winning amounting to P11,000 = Exempt PCSO winning amounting to P11,000= Exempt
D. PCSO winning amounting to P11,000 = Exempt PCSO winning amounting to P11,000= 25% FWT

75. The FWT for interest income from a Depository Bank under the Expanded Foreign Currency Deposit System is
RC NRC RA NRA-ETB NRA-NETB
A. 15% 15% 15% Exempt Exempt
B. 15% Exempt 15% Exempt Exempt
C. 15% Exempt 7.5% Exempt Exempt
D. 75% Exempt 7.5% Exempt Exempt

76. The FWT for interest income from a Depository Bank under the Expanded Foreign Currency Deposit System is
DC RFC NRFC
A. 15% 7.5% Exempt
B. 15% 15% Exempt
C. 15% Exempt Exempt
D. 75% 7.5% Exempt

77. The capital gains tax on sale of unlisted shares is:


RC NRC RA NRA-ETB NRA-NETB
A. 15% 15% 15% 15% 15%
B. 15% Exempt 15% Exempt Exempt
C. 15% Exempt 7.5% Exempt Exempt
D. 75% Exempt 7.5% Exempt Exempt
78. The capital gains tax on sale of unlisted shares is:
DC RFC NRFC
A. 15% 5% ; 10% 5% ; 10%
B. 15% 15% Exempt
C. 15% Exempt Exempt
D. 75% 7.5% Exempt

79. S1 - Preferential tax treatment shall not apply for employees of ROHQ, RHQ, OBU, and Petroleum service contractors and
subcontractors which registered with the Securities and Exchange Commission beginning 1 January 2018.
S2 - Amount of exempt 13th month pay and other benefits is increased to PHP90,000
A. True ; True
B. True ; False
C. False; True
D. False; False

80. Which of the following is subject to a corporate tax of 32%


A. PCSO
B. Government Service Insurance System
C. Social Security System
D. Local water district

81. Under the TRAIN Law, the following de minimis benefit ceiling increased, which is incorrect
A. Uniform allowance – increased to P5,000
B. Uniform allowance – increased to P6,000
C. Rice allowance – increased to P2,000 per month
D. Medical assistance to dependent – increased to P2,000 per year

(Items 82 and 83)In 2014, George James, single, supports the following: Earl, his significant other; Elirie, a legally adopted child

LEAD TAX Pre Week Page 10 of 28


LEARNING ADVANCEMENT REVIEW CENTER LEAD
who became 21 years old during the year; Vanessa, an 18-year old niece and Gorgonio, a senior citizen who is not related to
him. He earned P2,500,000.00 from his beauty parlor and received P250,000.00 as Christmas gift from his spinster aunt. He
had no other receipts for the year. The cost of services of the parlor was P500,000.00 and the expenses for the operation of his
beauty parlor amounted to P250,000.00.

82. The Output VAT for 2014 is:


A. P330,000
B. P300,000
C. P240,000
D. None

83. The taxable net income for 2014 if he avails of the optional standard deduction is:
A. P1,675,000.00
B. P1,500,000.00

C. P1,425,000.00
D. None

84. Which of the following income is not from a related trade, business or activity of a domestic proprietary educational
institution?
A. Income from the hospital where medical graduates are trained for residency
B. Income from the canteen situated within the school campus
C. Income from bookstore situated within the school campus
D. Income from rent of available office spaces in one of the school buildings

85. A tax imposed in the nature of a penalty to the corporation to deter tax avoidance of shareholders who avoid paying the
dividends tax on the earnings distributed to them by the corporation.
A. Minimum corporate income tax
B. Optional corporate income tax
C. Improperly accumulated earnings tax
D. Capital gains tax

86. Which of the following will not result to capital gains or losses?
A. Gains or losses on account of failure to exercise a privilege or option to buy or sell property
B. When a corporation distributes all of its assets in complete liquidation
C. When a partner retires or when the partnership is dissolved
D. Gains or losses from sale of office equipment

87. (Phil. CPA) Mr. Santiago purchased a life annuity for P100,000 which would pay him P10,000 a year. The life expectancy of
Mr. Santiago was 12 years. Which of the following would Mr. Santiago be able to exclude from his gross income?
A. P120,000
B. P100,000
C. P20,000
D. P10,000

88. The monetary value of the following housing privilege shall be fifty percent (50%) of the value of the benefit, except which
of the following?
A. If the employer leases a residential property for the use of his employee and the said property is the usual place of
residence of the employee
B. If the employer owns a residential property and the same is assigned for the use of his employee as his usual place
of residence
C. If the employer purchases a residential property on installment basis and allows his employee to use the same as
his usual place of residence
D. If the employer acquires a residential property and transfers the ownership to his employee without consideration.

89. NOLCO shall be availed of on a:


A. “first-in, first-out” basis.
B. “last-in, first-out” basis.
C. “weighted average” basis.
D. none of the choices.

90. Research and development expenses treated as deferred expenses shall be allowed as deduction ratably distributed over a
period of not less than:
A. 60 months beginning with the month ending the taxpayer’s taxable year.
B. 60 months beginning with the month in which the taxpayer first realizes benefits from such expenditures.
C. 30 months beginning with the month in which the taxpayer first realizes benefits from such expenditures.
D. 30 months beginning with the month in which the taxpayer first realizes benefits from such expenditures.

93. Which of the following may NOT be allowed to claim OSD in lieu of the itemized deductions?
A. Taxable estates and trusts
B. Domestic corporation
C. Resident foreign corporation
D. Non-resident foreign corporation

94. First statement: The additional exemption for dependent shall be claimed by only one of the spouses in the case of
married individuals.

LEAD TAX Pre Week Page 11 of 28


LEARNING ADVANCEMENT REVIEW CENTER LEAD

Second statement: The husband is the proper claimant of the additional exemption for qualified dependent children unless he
explicitly waives his right in favor of his wife.
A. True ; True
B. True ; False
C. False; True
D. False ; False

95. How much is the documentary stamp tax on deeds of sale and conveyance of real property?
I - Fifteen pesos (P15.00) when the consideration, or value received or contracted to be paid for such realty after making
proper allowance of any encumbrance, does not exceed One thousand pesos (P1,000) fifteen pesos (P15.00)
II - Fifteen pesos (P15.00) for each additional One thousand Pesos (P1,000), or fractional part thereof in excess of One
thousand pesos (P1,000) of such consideration or value
A. True ; True
B. True ; False
C. False; True
D. False ; False

96. There shall be allowed a deduction from gross income for entertainment, amusement and recreation expense in an amount
equivalent to the actual entertainment, amusement and recreation expense paid or incurred within the taxable year by the
taxpayer, but in no case shall such deduction exceed:
I - 0.50 % of net sales (gross sales less sales returns or allowances and sales discounts) for taxpayers engaged in sale of
goods or properties.
II - 1 % of net revenue (gross revenue less discounts) for taxpayers engaged in sale of services, including exercise of
profession and use or lease of properties.
A. True ; True
B. True ; False
C. False; True
D. False ; False

97. The taxpayer’s otherwise allowable deduction for interest expense shall be reduced by how much starting January 1, 2009?
A. Forty-five percent (45%) of the interest income subjected to final tax
B. Forty-two percent (42%) of the interest income subjected to final tax
C. Thirty-three percent (33%) of the interest income subjected to final tax
D. Twenty-five percent (25%) of the interest income subjected to final tax

98. At the option of the taxpayer, interest incurred to acquire property used in trade or business or exercise of profession may
be:
I - allowed as a deduction.
II - treated as a capital expenditure.
A. True ; True
B. True ; False
C. False; True
D. False ; False

99. Interest incurred or paid by the taxpayer on all unpaid business-related taxes:
I - shall be fully deductible from the gross income.
II - shall not be subject to reduction by an amount equal to certain percentage of the interest income subject to final tax.
A. True ; True
B. True ; False
C. False; True
D. False ; False

100. (Phil. CPA) Mrs. Evangelista owns a parcel of land worth P500,000 which she inherited from her father in 1990 when it was
worth P300,000. Her father purchased it in 1980 for P100,000. If Mrs. Evangelista transfers this parcel of land to her wholly
owned corporation in exchange for shares of stock of said corporation worth P450,000, Mrs. Evangelista’s deductible loss is:
A. Zero
B. P50,000.
C. P150,000.
D. P350,000.

PART 2
Tax on Transfer of Real Property Ownership

1. Which of the following is not subject to tax on transfer or real property ownership?
a. Sale or donation of real properties
b. Transfer of real properties through testate and intestate succession
c. Transfer of real property in complete liquidation by a corporation of its real properties to its stockholder
d. Transfer of ownership of real property pursuant o the Comprehensive Agrarian Law (CARP Law)

2. The tax base of the local tax on transfer of real property ownership is:
a. Total consideration involved in the acquisition of the property
b. Fair market value in case the monetary consideration involved in the transfer is not substantial
c. Total consideration involved in the acquisition of the property or fair market value in case the monetary
consideration involved in the transfer is not substantial, whichever is lower
d. Total consideration involved in the acquisition of the property or fair market value in case the monetary
consideration involved in the transfer is not substantial, whichever is higher

LEAD TAX Pre Week Page 12 of 28


LEARNING ADVANCEMENT REVIEW CENTER LEAD
3. When does the local tax on transfer or real property ownership accrue?
a. From the effective date of transfer of ownership of title every real property
b. Date of registration of the corresponding Deed of Conveyance
c. Any time within (6) months after the transfer of real property
d. Anytime the taxpayer signifies his desire to pay the local tax

4. Who shall have authority to collect the local transfer tax?


a. Province where the property is located
b. Residence of the transfer where the property is located
c. Principal office of the transferor where the property is located
d. Residence of the transferee where the property is located

5. An individual taxpayer sold his residential house and lot for P5,000,000 (fair market value was P4,000,000).

Question 1 – How much is the capital gains tax?


a. P300,000 b. P200,000 c. P75,000 d. None of the choices

Question 2 – How much is the documentary stamp tax?


a. P300,000 b. P200,000 c. P75,000 d. None of the choices

Question 3 – How much is the local transfer tax?


a. P75,000 b. P50,000 c. P25,000 d. None of the choices

Tax on business of Printing and publication

6. Which of the following is not subject to tax on business of printing and publication?
a. Printing and/or publication of books
b. Printing and/or publication of cards, posters, leaflets and handbills
c. Printing and/or publication of certificate , receipts, pamphlets and others of similar nature
d. Printing and/or publication of books and other reading materials prescribed by the Department of Education as
school texts or references

7. The rate of the tax on business or printing and publication must be fixed by a revenue ordinance and:
a. Should not be more than 50% of 1%
b. Should be more than 50% of 18%
c. Should be 1/20 of 1%
d. Should be more than 5%

8. Statement I: The tax on business of printing and publication shall accrue on the first day of January of each year.

Statement II: The tax on business of printing and publication shall be paid within the first 20 days of January or of each
subsequent quarter, as the case may be.

Statement III: The Sanggunian concerned may, for a justifiable reason or cause, extend the time for payment of this tax,
without surcharges or penalties, but only for a period not exceeding 6 months.

a. Only I and II are true


b. Only I and III are true
c. I, II and III are true
d. I, II and III are not true

9. Considering that the tax on business of printing and publication is in the nature of a business tax, which of the following
rules shall apply as far as the situs of the tax is concerned?

Rule 1: If the taxpayer maintains operates a branch of sales outlet where the sale or transaction is made and recorded, the
tax on such sale or transaction should be paid to the province where such branch or sales outlet is located.

Rule 2: In case where there is no branch or sales outlet in the province where the sale or transaction is made and recorded,
the tax on such sale or transaction should be paid to the province where the principal office is located.

a. Only Rule 1 applies


b. Only Rule 2 applies
c. Rule 1 and Rule 2 apply
d. Rule 1 and Rule 2 do not apply
e.
10. The following data are taken from the books of your client, single, in preceding calendar year:

Gross receipts from printing of books prescribed


by the Department of Education
as school texts or references P500,000
Gross receipts, printing of other books 200,000
Gross receipts, printing of cards, posters, leaflets and handbills 300,000
Cost of services 250,000
Deductible expenses 100,000

Question 1 – How much is the tax on business of printing and publication?


a. P5,000
b. P2,500
c. P2,003000
d. None of the choices

LEAD TAX Pre Week Page 13 of 28


LEARNING ADVANCEMENT REVIEW CENTER LEAD
Question 2 – How much was taxable net income in the preceding year?
a. P650,000
b. P100,000
c. Exempt from income tax
d. None of the choices

Franchise Tax

11. The rate of local franchise tax must be fixed by a revenue ordinance and :
a. Should not be more than 50% of 1%
b. Should be more than 50% of 1%
c. Should be 6%
d. Should be 10%

Tax on the Extraction of Sand, Gravel and Other Quarry Resources

12. The permit to extract and gravel and other quarry resources may only be issued pursuant to the revenue ordinance of the
Sangguniang Panlalawigan by the :
a. Provincial Governor.
b. Provincial Assessor.
c. Provincial Engineer.
d. Provincial Director of PNP.

13. Considering the nature of the quarrying activity, who may prescribe the time, manner, terms and conditions for the
payment of tax?
a. Provincial Governor.
b. Provincial Assessor.
c. Provincial Engineer.
d. Provincial Panlalawigan.

14. The Situs of the tax on extraction of sand, gravel and other quarry resources is :
a. Province where, the principal place of office of the taxpayer is located.
b. Province where the residence of the taxpayer is located.
c. Province where the quarry resources are extracted.
d. Any of the three province mentioned above.

15. Rakobato Company is engaged in quarrying business. It extracts sand and gravel in the Barangay Nabangig. Municipality of
Palanas, Province of Masbate. During a particular quarter, it extracted 500,000 cubic meters of sand from a public land
the fair market of which is P750,000. It also 200,000 cubic meters with fair market value of P250,000 from a quarry it
privately owns. How much tax on extraction will it be liable?
a. P100,000
b. P75,000
c. P50,000
d. None of the choices

16. Continuing with the preceding number, how shall the tax on extraction be distributed?
a. Nabangig, P30,000; Palanas P22,500; Masbate, P22,500
b. Nabangig, P25,000; Palanas P25,000; Masbate, P25,000
c. Nabangig, P40,000; Palanas P17,500; Masbate, P17,500
d. Nabangig, P0; Palanas P0; Masbate, P75,000

Professional Tax
17. Who among the following shall not be required to pay professional tax?
a. Person engaged in exercise or practice of his profession
b. Person who practice require government examination prior to their exercise or practice
c. Professionals exclusively employed by the government
d. None of the choice

Local Amusement Tax


18. Which among the following shall be subject both Value-Added Tax and the local Amusement Tax?
a. Theaters and cinemas
b. Cockpits
c. Concert halls
d. Circuses

Community Tax
19. Mr. and Mrs. A, resident citizens, had the following data for the calendar year:
Mr. A
Salaries and bonuses P500,750
Assessed value of land in USA 2,000,000
Assessed value of land in Manila 1,000,000
Mrs. A
Gross receipts from business P725,700
Zonal value of apartment house 3,000,000
Assessed value of apartment house 1,500,000
Income from apartment house 600,000
How much would be the total additional community tax due in the succeeding year aside from the P5 basic community tax
of each spouse?
a. P4,236
b. P2,500

LEAD TAX Pre Week Page 14 of 28


LEARNING ADVANCEMENT REVIEW CENTER LEAD
c. P1,826
d. P1,226

20. ABC corporation, domestic, had the following selected data in the preceding year:
Cash sales P3,390,000
Cost of sales 1,400,000
Operating expenses 900,000
Dividend received from a domestic corporation 50,000
Assessed value of land 500,000
FMV of land 1,000,000
Assessed value of building 700,000
Assessed value of machinery-Philippines 1,500,000
Assessed value of machinery-USA 2,000,000
How much would be the total basic and additional community taxes in the current year?
a. P2,956
b. P2,456
c. P1,376
d. P1,080

21. (Local Business Taxes) The following data are taken from the books of a VAT-registered manufacturer of cigarettes:
Gross sales P4,900,000
Sales returns 500,000
Sales discount, determinable at the time of sale 100,000
Excise tax 150,000
Value-added tax 540,000
Local tax passed-on to customers 100,000

For local business tax purposes, the taxable amount is:


a. P5,000,000 c. P3,710,000
b. P4,500,000 d. None of the choices

22. (Real Property Tax) A residential land is located in Metro Manila. Its fair market value is P5,000,000.

Question 1- How much is the basic real property tax, if any?


a. P30,000 c. P10,000
b. P20,000 d. None of the above

Question 2- How much is the special educational fund (SEF), if any?


a. P30,000 c. P10,000
b. P20,000 d. None of the choices

23. (Real Property Tax) An agricultural land is located in the province. Its fair market value is P2,000,000.

Question 1 – How much is the basic real property tax, if any?


a. P16,000 c. P4,000
b. P8,000 d. None of the choices

Question 2 – How much is the Special Educational Fund (SEF), if any?


a. P16,000 c. P4,000
b. P8,000 d. None of the choices

24. (Real Property Tax) A commercial land located in one of the cities in Metro Manila. Its fair market value is P10,000,000.

Question 1 – How much is the basic real property tax, if any?


a. P150,000 c. P50,000
b. P100,000 d. None of the choices

Question 2 – How much is the Special Educational Fund (SEF), if any?


a. P150,000 c. P50,000
b. P100,000 d. None of the choices

25. (Real Property Tax) A residential building located in one of the cities in Metro Manila has a fair market value of P10,000,000.
The city ordinance fixed the actual assessment level at 60%. How much is the real basic property tax, excluding the Special
Educational Fund (SEF)?
a. P120,000 c. P60,000
b. P100,000 d. None of the choices

Assessment Levels of Building


Class Fair Market Value Rate*
Residential P175,000 to P10,000,000 plus 0% to 60%
Agricultural P300,000 to P2,000,000 plus 25% to 50%
Commercial or Industrial 300,000 to P10,000,000 plus 30% to 80%
Timberland P300,000 to P2,000,000 plus 45% to 70%

*Actual assessment level fixed by ordinance

26. (Real Property Tax) An agricultural machinery located in the province has fair market value of P3,000,000. How much is the
real basic property tax, excluding Special Educational Fund (SEF)?
a. P12,000 c. P4,000

LEAD TAX Pre Week Page 15 of 28


LEARNING ADVANCEMENT REVIEW CENTER LEAD
b. P8,000 d. None of the choices

Assessment Levels of Machinery


Class Maximum Assessment Level
Residential 50%
Agricultural 40%
Commercial 80%
Industrial 80%

27. (PEZA) The ABC Company is a PEZA-registered manufacturer entitled to Income Tax Holiday (ITH) incentive for CY 2018. It is
the company’s fourth year of operations.
The following information pertain to the CY 2018 activities of the company:
Registered Unregistered
activities activities
Gross sales P121,700,000 P10,425,000
Cost of sales 103,400,000 7,297,500
Other income (net gain on disposal of office PPE) 550,500
Operating expenses of P9,890,100 (use 90%-10% allocation between registered and
unregistered activities)

Creditable withholding taxes (CWTs) from the first three (3) quarters amounted to P98,000 (including CWTs of P10,000 dated 2019)
while CWTs for the fourth quarter total P33,600 (excluding CWTs not in the name of ABC Company)

Question 1- How much is the 2% Minimum Corporate Income Tax (MCIT)?


a. P439,560 c. P62,550
b. P73,560 d. None of the choices

Question 2- How much is the 30% Regular Corporate Income Tax(RCIT)?


a. P1,103,400 c. zero
b. P806,697 d. None of the choices

Question 3- How much is the income tax still due?


a. P685,097 c. zero
b. P675,097 d. None of the choices
Regular Corporate Income Tax(RCIT) P806,697
Less: Creditable withholding taxes, first 3 quarters (88,000)
Creditable withholding taxes, fourth quarter (33,600)
Income tax still due P685,097

Question 4- Assuming the company is entitled to the 5% gross income tax (GIT), how much is the tax due?
a. P915,000 c. zero
b. P469,946 d. none of the choices

Question 5- Using the information in question no.4, determine the income tax still due to the BIR?
a. P1,721,697 c. P1,234,097
b. P1,600,000 d. None of the choices

Tariff and Customs Duties

29. A foreign country imposed very high and unreasonable duties on Philippine goods entering its jurisdiction without, however,
imposing the same high rates on goods coming from other countries. The remedy of the Philippine government in this situation is to
impose:
a. Countervailing duty
b. Anti-dumping duty
c. Marking duty
d. Discriminatory duty

30. A large quantity of goods were imported from a foreign country into the Philippines at a very low export prices that are much
less than their normal value, which threatens our domestic industry. The remedy of the Philippine government in this situation is to
impose:
a. Countervailing duty
b. Anti-dumping duty
c. Discriminatory duty
d. None of the foregoing

31. The amount that may be imposed on an ocean vessel for mooring at a pier in the country is:
a. Harbor fee
b. Wharfage fee
c. Berthing charge
d. None of the foregoing

32. The amount that may be imposed on vessels engaged in domestic trade to temporarily anchor at any port in the Philippines is:
a. Harbor fee
b. Arrastre charge
c. Lay up fee
d. None of the foregoing

33. The amount imposed on the owner or operator of a vessel for each entrance into or departure from a port of entry in the
Philippines is:
a. Harbor fee
b. Wharfage due

LEAD TAX Pre Week Page 16 of 28


LEARNING ADVANCEMENT REVIEW CENTER LEAD
c. Berthing charge
d. None of the foregoing

34. Which of the following is not absolutely banned from being imported into the Philippines?
a. Gambling devices
b. Treasonous articles
c. Obscene and immoral articles
d. None of the foregoing

35. Where the Collector of Customs has rendered a decision in a protest case that is adverse to the importer/owner, the latter may:
a. Appeal to the Commissioner of Customs within 15 days from the expiration of the 30-day period within which the Collector of
Customs must render his decision.
b. Appeal to the Commissioner of Customs within 15 days from receipt of the decision of the Collector of Customs.
c. Appeal to the Court of Tax Appeals within 30 days from receipt of the decision of the Collector of Customs.
d. None of the foregoing

36. Which of the following will render the determination of the customs duties, fees and charges by the Collector of Customs final
and conclusive?
a. Protest is filed simultaneously on the date in which payment is made by the importer/owner.
b. Protest is filed on the 15th day following the date in which payment is made by the importer/owner.
c. Appeal of the decision of the Collector of Customs to the Court of Tax Appeals is filed on the 15th day following the date in which
the decision is received by the importer /owner.
d. None of the foregoing

37. Which of the following statements is correct where the Commissioner of Customs affirmed the decision of the Collector of
Customs ordering the forfeiture of imported articles?
a. The importer/owner appeals tot he Court of Tax Appeals within 30 days from receipt of the adverse decision of the Commissioner
of Customs.
b. The importer/owner appeals to the Secretary of Finance within 30 days from receipt of the adverse decision of the Commissioner
of Customs.
c. The importer/owner appeals to the Secretary of Justice within 30 days from receipt of the adverse decision of the Commissioner
of Customs.
d. None of the foregoing

38. Which of the following statements is correct where the Collector of Customs rendered a decision against the government in a
seizure case involving an amount of P500,000?
a. Automatic review by the Commissioner of Customs
b. Automatic review by the Secretary of Finance
c. Automatic review by the Secretary of Justice
d. None of the foregoing

Local Government Tax Collecting Units

39. A resident of Davao City who does not agree with the real property tax assessment on his real property located in Pili,
Camarines Sur may file an appeal on the disputed assessment with the
a. Provincial Board of Assessment Appeals
b. Revenue District Officer
c. Court of Tax Appeals
d. City Board of Assessment Appeals

40. Not a party in the collection of real property tax in a city


a. Punong Barangay
b. City Mayor
c. City Treasurer
d. City Assessor

41. Which public office is not involved in the collection of community tax?
a. Provincial Treasurer’s Office
b. City Treasurer’s Office
c. City Assessor’s Office
d. Office of the Punong Barangay

42. A foreign corporation who wants to invest in the Philippines may register with the following, except
a. Assessor’s office where the principal office is located
b. Board of Investment
c. Bureau of Internal Revenue
d. Philippine Economic Zone Authority

43. The City Board of Assessment Appeals shall be composed of the following except
a. City Assessor
b. City Engineer
c. City Prosecutor
d. Registrar of Deed

PWDs/Senior Citizens

44. Masagana Corporation provides 20% discount to PWDs. It recorded the following during the year:

LEAD TAX Pre Week Page 17 of 28


LEARNING ADVANCEMENT REVIEW CENTER LEAD
Regular Customers PWD Total
Receipts P8,000,000 P1,000,000 P9,000,000
Cost of services 3,000,000 2,000,000 5,000,000
Other deductible expenses 1,000,000 1,000,000 2,000,000

Question 1: The amount of gross receipts to be reported is


a. P1,000,000 b. P1,250,000 c. P9,000,000 d. P9,250,000

Question 2: The taxable net income is


a. P750,000 b. P1,000,000 c. P1,750,000 d. P2,000,000

45. Masagana Corporation provides 25% discount to senior citizen. It recorded the following during the year:
Regular Senior Citizen Total
Gross sales P8,000,000 P2,000,000 P10,000,000
Cost of sales 5,000,000 1,000,000 6,000,000
Other deductible expenses 1,000,000 1,000,000 2,000,000

Question 1: The amount of gross sales to be reported is


a. P8,000,000 b. P9,500,000 c. P9,600,000 d. P10,000,000

Question 2: The taxable net income is


a. P1,750,000 b. P1,600,000 c. P1,800,000 d. P1,500,000

Documentary stamp tax

46. Effect of failure to affix a documentary stamp which is required by law, except
a. Shall not be recorded in the proper registry
b. Shall not be admitted as evidence in ay court
c. Shall not be valid
d. No notary public or other officer authorized to administer oaths shall add his jurat or acknowledgement

47. Which of the following is correct documentary stamp tax rate?


I. The original issue of shares P2 on each P200 of the par value of shares of stock

If without par value, based on the actual consideration


II. Sales, agreement to sell, deliveries or P1.50 on each P200 of the par value.
transfer of shares of stock
If without par value, the DST shall be 50% of the DST paid upon the original issue of
stock.
III. Bonds, debentures, certificate of stock Same as original issue of shares (P2/200)
or indebtedness issued in foreign country
IV. Debt instruments P1.50 on each P200
V. Bills of exchange P0.60 on each P200
VI. Real property P15 for every P1,000
a. I and II
b. I, II, and VI
c. II, IV and V
d. I, II, III, IV, V and VI

Excise tax

48. To improve her body shape, Miss Y decided to undergo liposuction procedure and sought the services of Doc. Vicky, a clinic
operated outside the hospital and owned by Belo Medical Group, Inc. Doc. Vicky charged Miss Y the amount of P50,000 inclusive
of 12% VAT but exclusive of excise tax for the service rendered.

Required:
1. Compute the excise tax.
2. Compute the amount to be collected from Miss Y.

Original price (inclusive of VAT) P50,000.00

Gross Receipts (net of 12% VAT) (P50,000 / 1.12) P44,642.85


Add:
5% excise tax (44,642.85 x 5%) 2,232.15
12% VAT (44,642.85+2,232.15) x 12% 5,625.00
Total amount to be Collected from Miss Y P52,500.00

In the books of Belo Medical Group, Inc.:

Cash 52,500
Excise Tax Expense 2,232.15
Excise Tax Payable 2,232.15
Output VAT 5,625.00
Service Income 46,875.00

49. To improve her body shape, Miss Y decided to undergo liposuction procedure and sought the services of Doc. Vicky, a clinic
operated outside the hospital and owned by Belo Medical Group, Inc. Doc. Vicky charged Miss Y the amount of P50,000 inclusive
of 12% VAT but inclusive of excise tax for the service rendered.

LEAD TAX Pre Week Page 18 of 28


LEARNING ADVANCEMENT REVIEW CENTER LEAD
Required:
1. Compute the excise tax.
2. Compute the amount to be collected from Miss Y.

Original price (inclusive of VAT and excise) P50,000.00

Gross Receipts (net of 12% VAT, inclusive of excise) (P50,000 / P44,642.85


1.12)

Gross Receipts (net of Excise and VAT) (44,642.86/105%) P42,517.01


Add:
5% excise tax (42,517.01 x 5%) 2,125.85
12% VAT (44,642.86 x 12%) 5,357.14
Total amount to be collected from Miss Y P50,000

50. Miss Sandara had invasive cosmetic procedure done by Doctor Sy, an individual practitioner operating a clinic inside a
hospital. The hospital bills Miss Sandara other fees (e.g. supplies and fees for the use of operating room and hospital facilities) in
the amount of P20,000, in addition to the fees charged by Doctor Sy of P50,000 (inclusive of 12% VAT, exclusive of excise) for the
service performed.

Required:
1. Compute the excise tax.
2. Compute the amount to be collected from Miss S.

Billings by Hospital (VAT Exempt) Note 1 P20,000.00


Add: 5% excise tax (20,000 x 5%) 1,000.00
Doctor’s Fee (50,000 / 112%) Note 2 44,642.86
Add: 5% excise tax (44,642.86 x 5%) 2,232.14
Add: 12% VAT (44,642.86 + 2,232.14) x 12% 5,625.00
Total Amount to be collected from Miss Sandara P73,500

Note 1: Medical, dental, hospital and veterinary services are exempt from value added tax under section 109 (G) of the NIRC, as
amended, except those rendered by professionals.

Note 2: Doctor Sy is presumed to be self-employed and his annual receipts exceeded the threshold for VAT of P3 Million.

In the books of Doctor Sy:


Accounts receivable – Hospital (44,642.86+5,625 – 4,464.28) P45,803.58
Excise tax expense 2,232.14
Prepaid tax (44,642.86 x 10% EWT) 4,464.28
Service fee (44,642.86 + 2,232.14) 46,875.00
Output VAT 5,625.00

Cash 45,803.58
Accounts receivable – Hospital 45,803.58

In the books of the Hospital:


Cash P21,000
Excise tax expense 1,000
Service income – Non-VAT P21,000
Excise tax payable 1,000

Cash 52,500
Payable to Doctor Sy 45,803.58
Excise Tax Payable for Doctor Sy 2,232.14
Expanded withholding tax payable – Prof. fee (44,642.86 x 10%) 4,464.28

The Hospital shall remit to the BIR the amount of P4,464.28 representing creditable income tax withheld from the fees charged by
Doctor Sy, in accordance with existing withholding tax regulations. In addition to this, the Hospital shall file a return of its monthly
gross receipts using BIR Form No. 2200-C and remit the 5% excise tax amounting to P3,232.14 based on the gross receipts (net of
12% VAT and 5% excise tax) collected from the client, together with the Monthly Summary of Cosmetic Procedures Performed as
required in Section 5.1 of RR 2-2019

51. (Excise tax on sweetened beverages – Carbonated Beverages) Dodo Manufacturing Corporation will remove from the
place of production 100 cases of Super Cola using HFCS and non-caloric sweetener. Each case contains 6 bottles of 1.5 liters each.

Required: Compute the excise tax to be paid before removal.

No. of cases 100


Multiplied by Number of bottles per case x 6
Total Number of bottles 600
Multiplied by content per bottle x 1.5L
Total Volume in Liters 900 L
Multiplied by Specific Tax Rate P12.00
Total Excise Tax to be paid before removal P10,800

LEAD TAX Pre Week Page 19 of 28


LEARNING ADVANCEMENT REVIEW CENTER LEAD
52. (Excise tax on sweetened beverages – Powdered Juice) Sweety Import Corp. will remove from customs custody 50
cases of Four Seasons Powdered Juice using caloric and non-caloric sweetener containing 144 packs by 25 grams. Each 25 grams
pack can make 1 Liter (per serving suggestion appearing on the label).

Required: Compute the excise tax to be paid before removal.

No. of Cases 50
Multiplied by no. of packs per case X 144
Total no. of packs 7,200
Multiplied by serving suggestion per pack in liters of volume X 1L
Total Volume in Liters 7,200 L
Multiplied by Specific Tax Rate X P6.00
Total Excise Tax to be paid before removal P43,200.00

53. True or False


I. NON-ESSENTIAL GOODS- is subject to Twenty percent (20%) based on the wholesale price or the value of importation used by
the Bureau of Customs in determining Tariff and Customs Duties, net of Excise and Value-Added taxes
II. SWEETENED BEVERAGES – Using purely coconut sap sugar purely steviol glycosides is exempt from excise tax.

a. True; False b. False; False c. False;True d. True; True


TARIFF AND CUSTOMS

54. 1st Statement: No regular court may intervene in assessment of real property taxes; it is the primary jurisdiction of the Board of
Assessors.
2nd Statement: No jurisdiction by court will lie against real property tax collection by the Local Government.
3rd Statement: No court may intervene in smuggling cases.
a. True, False, True
b. True, True, False
c. False, True, False
d. False, False, True

55. A device or scheme resorted to enable merchandise affected by taxes to be exported and later sold in foreign countries under the
same terms as if it is not taxed at all is called?
a. Import Entry
b. Manifest
c. Drawings
d. Drawback

56. The following are the functions of the Bureau of Customs, except?
a. Assessment and collection of revenues from imported articles and all other impositions under the Tariff and Customs Code.
b. Control smuggling and related frauds.
c. Supervision and control over the entrance and clearance of vessels and aircrafts.
d. To investigate the operation of tariff and customs laws, including their relation to the national revenues, their effects upon
industries and labor of the country and to submit reports of its investigation.

57. Julia an importer protested an assessment and. classification by the collector of customs. However, the collector of customs
denied Julia's protest. Hence, Julia went to appeal adverse decision by the collector of customs with the Commissioner of Custom,
which of the following statement is correct in case the Commissioner decides to favour or deny Julia?

I. Adverse decision against Julia appeal to Court of Tax Appeal.


Ii. Adverse decision against the government appeal to Court of Tax Appeal.
III. Adverse decision against Julia appeal to Secretary of Finance.
IV. Adverse decision against the government appeal to the Secretary of Finance.

a. I and II
b. III and IV
c. I and IV
d. I, II, III, and IV

58. Which of the following is a special duty?


a. Ad Valorem
b. Specific
c. Compounding Duty
d. Dumping Duty

PREFERENTIAL TAXATION

59. The National Economic Development Authority in coordination with Board of Investment gave tax incentives to preferred areas of
activities, except?
a. Business Process Outsourcing
b. Electronic industry
c. Renewable energy
d. Medical tourism

60. Which is not a tax incentives given to Board of Investment registered enterprises?
a. Income tax holiday
b. Exemption from taxes and duties on imported spare parts
c. Credits and additional deductions from taxable income
d. Exemption from business permit and real property taxes

LEAD TAX Pre Week Page 20 of 28


LEARNING ADVANCEMENT REVIEW CENTER LEAD
61. The Philippines adheres to promote small scale business as essential to country's economic development since the government
gave tax exemption for incomes generated from its activities by Micro Business Enterprises. What, is therefore, a "Micro Business
Enterprise?"
a. Any business engaged in production, processing or manufacturing of products or commodities, including agro-processing,
trading and services whose total assets excluding loans and lands not exceeding Php3,000,000.
b. Any business engaged in production, processing or manufacturing of products or commodities, including agro-processing,
trading and services whose total assets including loans and excluding lands not exceeding Php3,000,000.
c. Any business engaged in production, processing or manufacturing of products or commodities, including agro-processing,
trading and services whose total capitalization not exceeding Php3,000,000.
d. Any business engaged in production, processing or manufacturing of products or commodities, including agro-processing,
trading and services whose total capitalization excluding lands not exceeding Php3,000,000.

FILING AND REMEDIES OF THE STATE AND TAXPAYER

62. Under the eFPS a return is deemed filed with the BIR after a sequence Number is generated. What is this sequence Number?
a. Filing Reference Number
b. Confirmation Number
c. Acknowledgement Number
d. Computerized Generated Number

63. In filing a protest with the Bureau of Internal Revenue, what remedies may be availed by the taxpayer simultaneously?
a. Reinvestigation or abatement;
b. Reinvestigation or compromise;
c. Reinvestigation or reconsideration;
d. Reinvestigation or review.

64. Which of the following is not a proper way to deliver a final assessment notice?
a. Registered Mail;
b. Electronic messaging;
c. Constructive Service;
d. Personal Service.

65. What are documents required to be filed with a Corporate Income Tax Return if the gross quarterly sales is between P45,000-
P48,000, except?
a. Certificate of Income payments not subjected to withholding tax;
b. Certificate of creditable withholding tax at source;
c. Duly approved tax debit memo;
d. Account information form/ or Certificate from independent CPA of latest audited financial statements;

66. What are documents required to be filed with a Corporate Income Tax Return if the gross quarterly sales is between P150,000
and above, except?
a. Certificate of Income payments not subjected to withholding tax;
b. Certificate of creditable withholding tax at source;
c. Duly approved tax debit memo;
d. Previous years ITR and proof of payment.

67. Is the estate required to file an income tax return?


Answer: YES, for income earned during the settlement period provided it is not exempted from income tax or subject to final
withholding tax.

Answer: No, decedents rights are transmitted to the heirs the moment of death, any income accruing after death shall inure to the
benefit and responsibility of the heirs.
a. Both statement are wrong;
b. Both statements are correct;
c. Only 1st statement is correct;
d. Only 2nd statement is correct.

68. Mr. Camilo Cruz your client asks you to file a tentative income tax return for his behalf because he is going abroad. However,
Mr. Cruz stay outside the Philippines is indefinite, so Mr. Cruz asks you when is his last day to make amendments to finalize his
Income Tax Return?
a. 3 years from the date of filing
b. 3 years from the date required by law
c. Anytime until the BIR has not yet commenced its investigation over said return
d. Before Mr. Cruz receives a Letter of Authority from the BIR

69. A BIR examiner issued subpoena duces tecum to a incorrigible taxpayer, nevertheless, did not heed to such subpoena. What
should the BIR examiner do if the statutory period is about to expire but the taxpayer is still not able to submit the requirements?
a. Issue a jeopardy assessment with accompanying formal letter of demand.
b. Issue a subpoena ad testificandum.
c. File criminal complaint for contempt with the Office of the Prosecutor
d. Issue a letter of List of Documentary Requirements.

70. What is income tax treaty?


a. Is convention or agreement for the avoidance of double taxation and the prevention of fiscal evasion with respect to taxes on
income (and on capital).
b. Is intended to promote international trade and investment in several ways, the most important of which is by allocating taxing
jurisdiction between the Contracting States so as to eliminate or mitigate double taxation of income.
c. Is intended to permit the Contracting States to better enforce their domestic laws so as to reduce tax evasion.
d. Is international agreement between two or more states (contracting states) for efficient enforcement and beneficial
implementation of tax laws.

71. What taxes are covered by Philippines Tax Treaties?

LEAD TAX Pre Week Page 21 of 28


LEARNING ADVANCEMENT REVIEW CENTER LEAD
I. Income tax
II. Business tax
III. Transfer tax
IV. Documentary stamp tax

a. Only I
b. Both I and II
c. All three I, II, and III
d. All of the above

72. A Domestic Corporation pays royalty to a Foreign Corporation but it withholds a tax lower than Philippine tax laws based on a
treaty. What documents need be presented to prove the imposition of a lower withholding tax?
a. Treaty law;
b. Memorandum of agreement between the local and international company;
c. Reciprocity clause;
d. Tax treaty relief application

73. The court of tax appeals is composing of:


a. 1 justice and 2 associate justices
b. 1 justice and 5 associate justices
c. 1 justice and 8 associate justices
d. 1 justice and 14 associate justices

74. As a Certified Public Accountant what would you tell if a domestic corporation asked you where to file first its financial statements
and income tax returns?
a. Securities and Exchange Commission
b. Local Municipal City Treasurer where the corporation has a principal address
c. Bureau of Internal Revenue
d. Authorized Accredited Agent Banks

75. Where to file income tax return?


a. Authorized Agent Bank where taxpayer is registered
b. Revenue District Office where taxpayer is registered
c. Local of City Treasurer where the taxpayer is domiciled
d. Commissioner of Internal Revenue

76. As a Certified Public Accountant what would you tell if a domestic corporation asked you where to file first its financial statements
and income tax returns?
a. Securities and Exchange Commission
b. Local Municipal City Treasurer where the corporation has a principal address
c. Bureau of Internal Revenue
d. Authorized Accredited Agent Banks

77. With regard to the preceding question, how many financial statement and tax return copies are filed with the Bureau of Internal
Revenue?
a. Duplicate
b. Triplicate
c. Quadruplicate
d. As many as required by the revenue officer

78. The guidelines and instructions require that BIR forms must be filled-out in _____
a. Duplicate b. Triplicate c. Quadruplicate d. Quintuplicate

79. The signature of the following persons must appear in the income tax return of a corporation, except
a. President b. General Manager c. Treasurer d. Independent certified public accountant

80. Which of the following is not contained in a statement of management's responsibility'?


a. The management is responsible for all information and representations contained in the Annual Income Tax Return for a
particular year
b. The management 'is responsible for all information and representations contained in the financial statements accompanying the
Annual Income Tax Return or Annual Information Return.
c. The management is not responsible for the information and representations contained in all the other tax returns filed for the
reporting period, such as VAT and/or percentage tax returns, withholding tax returns, documentary stamp tax returns, and any and
all other tax returns.
d. The management is responsible for all information and representations contained in all the other tax returns filed
for the reporting period.

81. Who of the following persons is not required to affix the signature in a Statement of Management Responsibility?
a. Individual taxpayer / President / Managing Partner
b. Chief Financial Officer
c. Chief Executive Officer
d. In the case of a foreign corporation with branch office in the Philippines, the manager who is in charge of its international
operations.

82. Mr. A wants to avail of the substituted filing of income tax return. Which of the following situations will disqualify him to avail of
such method of filing the return?
a. Receiving purely compensation income regardless of amount.
b. His spouse is disqualified and they are filing their tax return jointly.
c. Working for only one employer for the calendar year.
d. Tax has been withheld correctly by the employer.

LEAD TAX Pre Week Page 22 of 28


LEARNING ADVANCEMENT REVIEW CENTER LEAD
83. ACCNTNT Roman Velez is a certified public accountant. Which of the situations below will disqualify him to avail of the
substituted filing of tax return?
a. Employed in the government with no other source of income.
b. Engaged in public practice.
c. Full-time accounting teacher.
d. Employed as staff auditor of a big audit firm.

84. It is a tax preparation software that allows taxpayers and ATAs to accomplish or fill-up tax forms offline.
a. Offline eBIRForms Package
b. Online eBIRForms System
c. Electronic BIR forms (eBIRForms)
d. Electronic Filing and Payment System

85. It is a filing infrastructure that accepts tax returns submitted online and automatically computes penalties for tax returns
submitted beyond due date.
a. Offline eBIRForms Package
b. Online eBIRForms System
c. Electronic BIR forms (eBIRForms)
d. Electronic Filing and Payment System

86. Under the eFPS a return is deemed filed with the BIR after a sequence number is generated. What is this sequence number?
a. Filing reference number
b. Confirmation number
c. Acknowledgement number
d. Computerized general number

87. ABC Corporation had a net income per books on December 31, 2017 of P220,000. Included in the net income are the following
items: Dividend income from a domestic corporation, P50,000, Interest on bank deposit (net of tax), P5,000; and, provision for bad
debts, P35,000 Based on the given data, the taxable income of ABC Corporation for the year ended December 31 , 2017 is —
a. P130,000 b. P200,000 c. P220,000 d. P165,000

88. Mr. Agapito your client asks you to file a tentative income tax return for his behalf because he is going abroad. However, Mr.
Agapito stay outside the Philippine is indefinite , so Mr. Agapito asks you when is his last day to make amendments to finalize his
income tax return?
a. 3 years from the date of filing
b. 3 years from the date required by law
c. Anytime until the BIR has not yet recommend its investigation over said return
d. Before Mr. Agapito receives a Letter of Authority from the BIR

89. When shall a tax-exempt organization file its annual information return?
a. On or before January 15 of each year
b. On or before February 15 of each year
c. On or before March 15 of each year
d. On or before April 15 of each year

90. Who shall initially determine whether there is an improper accumulation of profit?
a. The Taxpayer
b. The Court of Tax Appeals
c. The Department of Justice
d. The BIR

91. Examination and inspection of books of accounts and other accounting records shall be done in the :
Taxpayer’s office Place of business In the of the BIR
A Yes Yes No
B No Yes No
C Yes No No
D Yes Yes Yes

Withholding taxes (at source, expanded or creditable withholding tax, final withholding taxes and withholding tax on
government payments)

92. Under this system, the amount of income tax withheld by the withholding agent is constituted as a full and final payment of the
income tax due from the payee on the said income.
a. Creditable withholding tax c. Global tax system
b. Final withholding tax d. Schedular tax system

93. Under this system, taxes withheld on certain income payments are intended to equal or at least approximate the tax due of the
payee on said income.
a. Creditable withholding tax c. Global tax system
b. Final withholding tax d. Schedular tax system

94. Which of the following statements is not correct?


a. Creditable withholding tax may refer to withholding tax on compensation or to withholding of business tax.
b. The withholding tax on business may refer either to VAT or to Percentage Tax.
c. Withholding tax on compensation may also refer to expanded withholding tax.
d. Creditable withholding tax is the other term for final withholding tax.

95. Withholding taxes are the primary liability of the


a. Earner or payee of income
b. Payor of income
c. Payee and payor of income

LEAD TAX Pre Week Page 23 of 28


LEARNING ADVANCEMENT REVIEW CENTER LEAD
d. Collecitng agency of the government.

96. XYZ is one of the Philippines registered top 20,000 private corporation. As the company’s accountant you where tasked to
account for all payments made local/resident suppliers. What percentage of withholding tax rates of income payments made by top
20,000 private corporations to their local/resident suppliers of goods?
a. 1% of the gross payments
b. 2% of the gross payments
c. 3% of the gross payments
d. 4% of the gross payments

97. Macky a non-stock and non-profit charitable institution pays for its regular repairs and maintenance of its facilities to Carmel
Manpower Service. As the accountant of the exempt entity, how much withholding tax will you impose to its supplier (Carmel) for the
repairs and maintenance expense?
a. Exempt from withholding tax
b. 1 % withholding tax on its gross payment
c. 2% withholding tax on its gross payment
d. 5 % withholding tax on its gross payment

98. ABC Domestic Corporation hired Velayo, Gorres and Santos accounting firm to conduct its annual audit for the year 2017. Ayer
Corporation paid VGS & Co., an engagement fee equivalent to Php 100 Million. Such payment to VGS & Co. shall be:
a. Exempt from withholding tax
b. Subject to 1 % withholding tax on its gross payment
c. Subject to 2 % withholding tax on its gross payment
d. Subject to 5 % withholding tax on its gross payment

99. Not subject to creditable withholding tax.


a. Rental of real property used in business.
b. Management and technical consultants
c. Lawyers
d. Sale of real property capital assets.

Accounting Period and Accounting Method

100. A corporation may change its taxable year to calendar or fiscal year in filing its annual income tax return, provided
a. it seeks prior BIR approval of its proposed change in accounting period
b. it simultaneously seeks BIR approval of its new accounting period.
c. it should change its accounting period two years prior to changing its taxable year.
d. its constitution and by-laws authorizes the change.

101. ABC Bookstore is using cash method of accounting in recognizing its taxable income and deductions. For the current year, ABC
decided to adopt the accrual method of recognizing income and expenses. The application for permission to change the method of
accounting employed and the basis upon which return is made shall be filed to the BIR within:
a. 90 days b. 60 days c. 30 days d. 15 days

102. Change in accounting period, if a taxpayer, other than an individual, from calendar year to fiscal year or from one fiscal year to
another, the application for permission to change must be filed no less than_____
a. 90 days b. 60 days c. 30 days d. 15 days

103. Which among the following taxpayers is required to use only the calendar year for tax purposes?
a. Partnership exclusively for the design of government infrastructure projects considered as practice of civil engineering.
b. Joint-stock company formed for the purpose of undertaking construction Projects
c. Business partnership engaged in energy operations under a service contract with the government
d. Joint account (cuentas en participacion) engaged in the trading of mineral ores.

104. A hybrid method of accounting is –


a. the method under which expenses in the production of crops are deducted in the year in which the gross income from the crop
has been realized.
b. a method under which the taxpayer reports his income and expenses by employing the combination of cash and accrual method.
c. also known as cash receipts and disbursements method.
d. a method in which the taxpayer reports income in the year it is earned.

105. The "all events test" refers to


a. A person who uses the cash method where all sales have been fully paid by the buyers thereof
b. A person who uses the installment sales method, where the full amount of consideration is paid in full by the buyer thereof
within the year of sale.
c. A person who uses the accrual method, whereby an expense is deductible for the taxable year in which all the events had
occurred which determined the fact of the liability and the amount thereof could be determined with reasonable accuracy.
d. A person who uses the completed method, whereby the construction project has been completed during the year the contract
was signed

106. Statement 1: Where donor's tax has been paid on property received by a minor from a living parent, income on such property
shall be included in the income tax return of the parent.

Statement 2. The income tax return of a disabled person maybe made by a person charged with the care of his property.

Statement1 Statement 2
a. True True
b. False False
c. False True
d. True False

Others

LEAD TAX Pre Week Page 24 of 28


LEARNING ADVANCEMENT REVIEW CENTER LEAD

107. Choose the correct answer: Tax avoidance –


(A) is a scheme used outside of those lawful means and, when availed of, it usually subjects the taxpayer to further additional civil
or criminal liabilities.
(B) is a tax saving device within the means sanctioned by law.
(C) is employed by a corporation, the organization of which is prompted more on the mitigation of tax liabilities than for legitimate
business purpose.
(D) is any form of tax deduction scheme, regardless if the same is legal or not.

108. Which of the following are NOT usually imposed when there is a tax amnesty?
(A) Civil, criminal, and administrative penalties
(B) Civil and criminal penalties
(C) Civil and administrative penalties
(D) Criminal and administrative penalties

109. Anne Lapada, a student activist, wants to impugn the validity of a tax on text messages.
Aside from claiming that the law adversely affects her since she sends messages by text, what may she allege that would strengthen
her claim to the right to file a taxpayer’s suit?
(A) That she is entitled to the return of the taxes collected from her in case the court nullifies the tax measure.
(B) That tax money is being extracted and spent in violation of the constitutionally guaranteed right to freedom of communication.
(C) That she is filing the case in behalf of a substantial number of taxpayers.
(D) That text messages are an important part of the lives of the people she represents.

110. Which of the following statement is NOT correct?


a) In case of doubt, statutes levying taxes are constructed strictly the government; 

b) The construction of a statute made by his predecessors is not binding upon the successor, if thereafter he becomes satisfied that
a different construction should be 
given; 

c) The reversal of a ruling shall not generally be given retroactive application, if said 
reversal will be prejudicial to the taxpayer;
d) A memorandum circular promulgated by the CIR that imposes penalty for violations 
of certain rules need not be published in a
newspaper of general circulation or official gazette because it has the force and effect of law. 


111. The commissioner of internal revenue issued a BIR ruling to the effect that the transaction is liable to income tax and value
added tax. Upon receipt of the ruling, a taxpayer does not agree thereto. What is his proper remedy?
(A) File a petition for review with the CTA within 30 days from receipt thereof.
(B) File a motion for reconsideration with the Commissioner of Internal Revenue
(C) File an appeal to the Secretary of Finance within 30 days from receipt thereof.
(D) File an appeal to the Secretary of Justice within 30 days from receipt thereof.

112. The BIR, through the Commissioner, instituted a system requiring tax payers to submit to the BIR a summary list of their
sales and purchases during the year, indicating the name of the seller or the buyer and the amount. Based in these lists, the BIR
discovered that in 2007 ABC Corp. did not declare these for income tax purposes as its reported gross sales for 2007 was only
P1000,000.00. Which of the following defences may XYZ Corp. interpose in an assessment against it by the BIR?
a) The BIR has no authority to obtain third party information to assess taxpayers;
b) The third party information is inadmissible as hearsay evidence;
c) The system requiring taxpayers to submit third party information is illegal violating the right to privacy;
d) Not one of the defences in choices (A), (B), and (C) are valid. This is so because the actuations of the BIR Commissioner are
within the ambit of his powers granted under the NIRC of 1997, specifically Sec.5 Power of the commissioner to obtain
information, and to summon/ examine and take testimony of persons and sec.6 power of the commissioner to make
assessments and prescribe additional requirements fot tax administration and enforcement.

113. In “Operation Kandado,” the BIR temporarily closed business establishments, including New Dynasty Corporation that failed to
comply with VAT regulations. New Dynasty contends that it should not be temporarily closed since it has a valid and existing VAT
registration, it faithfully issued VAT receipts, and filed the proper VAT returns. The contention may be rejected if the BIR
investigation reveals that:
A. the taxpayer has not been regularly filing its income tax returns for the past 4 years.
B. the taxpayer deliberately filed a false and fraudulent return with deliberate intention to evade taxes.
C. the taxpayer used falsified documents to support its application for refund of taxes.
D. there was an understatement of taxable sales or receipts by 30% or more for the taxable quarter.

114. The Commissioner of Internal revenue may NOT inquire into the bank deposits of a taxpayer, except:
a. when the taxpayer files a fraudulent return.
b. when the taxpayer offers to compromise the assessed tax based erroneous assessment.
c. when the taxpayer offers to compromise the assessed tax based on financial incapacity to pay and he authorized the
Commissioner in writing to look into his bank records.
d. when the taxpayer did not file his income tax return for the years.

115. In January 2016, the BIR issued a ruling that Clemen’s vodka imports were not subject to increased excise tax based on his
claim that his net retail price was only P200 per 750 milliliter bottle. This ruling was applied to his imports for May, June and July
2016. In September 2016, the BIR revoked its ruling and assessed him for deficiency taxes respecting his May, June and July 2016
vodka imports because it discovered that his net retail price for the vodka was P250 per bottle from January to September 2016.
Does the retroactive application of the revocation violate Clemen’s right to due process as a taxpayer?
A. Yes, since the presumption is that the BIR ascertained the facts before it made its ruling.
B. No, because he acted in bad faith when he claimed a lower net retail price than what he actually used.
C. No, since he could avail of remedies available for disputing the assessment.
D. Yes, since he had already acquired a vested right in the favourable BIR ruling.

116. ABS Corporation is a PEZA-registered export enterprise which manufactures cameras and sells all its finished products abroad.
Which statement is NOT correct?
a. ABS Corporation is subject to the 5% final tax on gross income earned, in lieu of all national and local taxes;
b. ABS Corporation is exempt from the 30% corporate income tax on the net income, provided it pays value added tax;
c. ABS Corporation is subject to the 30% corporate income tax on net income;

LEAD TAX Pre Week Page 25 of 28


LEARNING ADVANCEMENT REVIEW CENTER LEAD
d. ABS Corporation is exempt from all national and local taxes, except real property tax.

117. Which among the following taxpayers is required to use only the calendar year for tax purposes?
a. Partnership exclusively for the design of government infrastructure projects considered as practice of civil engineering.
b. Joint stock company formed for the purpose of undertaking construction projects.
c. Business partnership engaged in energy operations under a service contract with the government.
d. Joint account (cuentas en participation) engaged in the trading of mineral ores.

118. A corporation may change its taxable year to calendar or fiscal year in filing its annual income tax return provided:
a. it seeks prior BIR approval of its proposed change in accounting period.
b. it simultaneously seeks BIR approval of its new accounting period.
c. it should change its accounting period two years prior to changing its taxable year.
d. its constitution and by-laws authorizes the change.

119. There is no taxable income until such income is recognized. Taxable income is recognized when the
a. Taxpayer fails to include the income in his income tax return.
b. Income has been actually received in money or its equivalent.
c. Income has been received, either actually or constructively.
d. Transaction that is the source of the income is consummated.

120. The "all events test" refers to:


a) A person who uses the cash method where all sales have been fully paid by the buyers thereof;
b) A person who uses the installment sales method, where the full amount of consideration is paid in full by the buyer thereof
within the year of sale;
c) A person who uses the accrual method, whereby an expense is deductible for the taxable year in which all the events had
occurred which determined the fact of the liability and the amount thereof could be determined with reasonable accuracy;
d) A person who uses the completed method, whereby the construction project has been completed during the year the
contract was signed.

121. In 2010, Mr. Platon sent his sister Helen $1 ,000 via a telegraphic transfer through the Bank of PI. The bank's remittance clerk
made a mistake and credited Helen with $1,000,000 which she promptly withdrew. The bank demanded the return of the mistakenly
credited excess, but Helen refused. The BIR entered the picture and investigated Helen. Would the BIR be correct if it determines
that Helen earned taxable income under these facts?
(A) No, she had no income because she had no right to the mistakenly credited funds.
(B) Yes, income is income regardless of the source.
(C) No, it was not her fault that the funds in excess of $1,000 were credited to her.
(D) No, the funds in excess of $1,000 were in effect donated to her.

122. Income from the performance of service is treated as income from within the Philippines, if:
a. The payment of compensation for the service is made in the Philippines;
b. The contract calling for the performance of service is signed in the Philippines;
c. The service is actually performed in the Philippines;
d. The recipient of service income is a resident of the Philippines.

123. Alain Descartes, a French citizen permanently residing in the Philippines, received several items during the taxable year. Which
among the following is NOT subject to Philippine income taxation?
(A) Consultancy fees received for designing a computer program and installing the same in the Shanghai facility of a Chinese firm.
(B) Interests from his deposits in a local bank of foreign currency earned abroad converted to Philippine pesos.
(C) Dividends received from an American corporation which derived 60% of its annual gross receipts from Philippine sources for the
past 7 years.
(D) Gains derived from the sale of his condominium unit located in The Fort, Taguig City to another resident alien.

124. Income from the performance of services is treated as income from within the Philippines, if:
a) The payment of compensation for the service is made in the Philippines;
b) The contract calling for the performance of services is signed in the Philippines;
c) The service is actually performed in the Philippines;
d) The recipient of service income is a resident of the Philippines.

125. For income tax purposes, the source of the service income is important for the taxpayer, who is a: (2012)
a) Filipino citizen residing in Makati City;
b) Non-resident Filipino citizen working and residing in London, United Kingdom;
c) Japanese citizen who is married to a Filipino citizen and residing in their family home located Fort Bonifacio, Taguig City;
d) Domestic corporation.

126. Guidant Resources Corporation, a corp. registered in Norway, has 50 MW electric power plant in San Jose, Batangas. Aside
from its income from its power plant, which among the following is considered as part of its income from sources within the
Philippines?
a. Gains from the sale to an Ilocos power plant of generators brought from the US.
b. Interests earned on its dollar deposits in a Philippine bank under the Expanded Foreign Currency Deposit System.
c. Dividends from a 2-year old Norwegian subsidiary with operations in Zambia but derives 60% of its gross income from the
Philippines.
d. Royalties from the use in Brazil of generator sets designed in the Philippines by its engineers.

127. PRT Corp. purchased a residential house and lot with a swimming pool in an upscale subdivision and required the company
president to stay there without paying rent; it reasoned out that the company president must maintain a certain image and be able
to entertain guests at the house to promote the company’s business. The company president declared that because they are
childless, eh and his wife could very well live in a smaller house. Was there a taxable fringe benefit?
a. There was no taxable fringe benefit since it was for he convenience of the employer and was necessary for its business.
b. There was a taxable fringe benefit since the stay at the house was for free.
c. There was a taxable fringe benefit because the house was very luxurious.
d. There was no taxable fringe benefit because the company president was only required to stay there and did not demand
free housing.

LEAD TAX Pre Week Page 26 of 28


LEARNING ADVANCEMENT REVIEW CENTER LEAD

128. Income from dealings in property (real, personal, or mixed) is the gain or loss derived:
a. only from the cash sales of property.
b. from cash and gratuitous receipts of property.
c. from sale and lease of property.
d. only from the sale of property.

129.An individual, who is a real estatedealer, sold a residential lot in Quezon Cityat a gain of P100,000.00 (selling price of
P900,000.0 and cost is P800,000.00). Thesale is subject to income tax as follows:
(A) 6% capital gains tax on the gain;
(B) 6% capital gains tax on the gross sellingprice or fair market value, whichever ishigher;
(C) Ordinary income tax at the graduatedrates of 5% to 32% of net taxableincome;
(D) 30% income tax on net taxable income.

130.A dealer in securities sold unlistedshares of stocks of a domestic corporationin 2016 and derived a gain of P1 Milliontherefrom.
The gain is:
(A) Taxable at 30% regular corporateincome tax based on net taxable income;
(B) Taxable at 5%/10% capital gains taxbased on net capital gain;
(C) Taxable at ½ of 1% stock transactiontax based on the gross selling price or fairmarket value, whichever is higher
(D) Exempt from income tax.

131.InMarch 2015 Tonette, who is fond of jewelries, bought a diamond ring forP750,000.00, a bracelet for P250,000.00, anecklace f
or P500,000.00, and a brooch forP500,000.00. Tonette derives income fromthe exercise of her profession as a licensedCPA. In Octob
er 2015, Tonette sold herdiamond ring, bracelet, and necklace foronly P1.25 million incurring a loss ofP250,000.00. She used the P1.
25 million tobuy a solo diamond ring in November 2009which she sold for P1.5 million inSeptember 2016. Tonette had no othertrans
action in jewelry in 2016. Whichamong the following describes the taximplications arising from the abovetransactions?
(A) Tonette may deduct his 2015loss only from her 2015professionalincome.
(B) Tonette may carry over anddeduct her 2015loss only fromher 2016 gain.
(C) Tonette may carry over anddeduct her 2015loss from her 2016professional income as wellas fromher gain.
(D) Tonette may not deduct her2015 loss from both her 2016professional income and her gain.

132. Keyrand, Inc., a Philippine corporation, sold through the local stock exchange 10,000 PLDT shares that it bought 2 years ago.
Keyrand sold the shares for P2 million and realized a net gain of P200,000.00. How shall it pay tax on the transaction?
a. It shall declare a P2 million gross income in its income tax return, deducting its cost of acquisition as an expense.
b. It shall report the P200,000.00 in its corporate income tax return adjusted by the holding period.
c. It shall pay 5% tax on the first P100,000.00 of the P200,000.00 and 10% tax on the remaining P100,000.00.
d. It shall pay a tax of one-half of 1% of the P2 million gross sales.

133. A resident Filipino citizen (not a dealer in securities) sold shares of stocks of a domestic corporation that are listed and traded
in the Philippine Stock Exchange.
a. The sale is exempt from income tax but subject to the ½ of 1% stock transaction tax.
b. The sale is subject to income tax computed at the graduated income tax rates of 5% to 32% on net taxable income.
c. The sale is subject to the stock transaction tax and income tax.
d. The sale is both exempt from the stock transaction tax and income tax.

134. Dimas owns a parcel of land worth P5,000,000.00 which he inherited from his father in 2011 when it was worth P3,000,000.00.
His father purchased the property in 2001 for P1,000,000.00.

A) If Dimas transfers this parcel of land to his wholly owned corporation, Dimasalang Corporation, in exchange for shares of stock of
said corporation worth P4,500,000.00, Dimas will have a taxable gain of:
(1) Zero
(2) P500,000.00
(3) P2,000,000.00
(4) P3,500,000.00
(5) P4,000,000.00
(6) None of the above

Choose one of the above answers and explain the reasons for your choice. (1986, dates and rewording supplied)

(4) P3,500,000.00. The exchange is not one solely in kind which is subject to tax-exemption. To be exempt, the
exchange must be for the purpose of obtaining control which is not present in this case because Dimas already controls
Dimasalang Corporation which he owns wholly.

B) Dimasalang Corporation in the above situation would have a taxable gain of:
(1) Zero
(2) P500,000.00
(3) P1,500,000.00
(4) P3,500,000.00
(5) None of the above.

Choose one of the above answers and explain the reasons for your choice. (1986, adapted and reworded)

SUGGESTED ANSWER:
(2) P500,000.00. Dimasalang parted with an asset worth P4,500,000.00 with another with a value of
P5,000,000.00. The exchange is not exempt because Dimas already controls the corporation. In a tax-free exchange,
the purpose must be for controlling the corporation.

C) As a result of the transaction above-mentioned, the aggregate basis of Dimas for all his Dimasalang Corporation shares is:
(1) P1,000,000.00
(2) P3,000,000.00
(3) P4,500,000.00
(4) P5,000,000.00

LEAD TAX Pre Week Page 27 of 28


LEARNING ADVANCEMENT REVIEW CENTER LEAD
(5) None of the above.

Choose one of the above answers and explain the reasons for your choice. (1986, adapted and reworded)

(2) P3,000,000.00. This is the fair market price or value of the property as of the date the property was acquired
through inheritance.

D) Going back to the transaction in question no. 2, if instead of inheriting the parcel of land from his father, Dimas received it as a
gift, the aggregate basis of Dimas for all his Dimasalang Corporation shares would be:
(1) P1,000,000.00
(2) P3,000,000.00
(3) P4,500,000.00
(4) P5,000,000.00
(5) None of the above.

Choose one of the above answers and explain the reasons for your choice. (1986, adapted and reworded)
(1) P1,000,000.00 which was the value at the hands of his father.

135. Passive income includes income derived from an activity in which the earner does not have any substantial participation. This
type of income is:
(A) usually subject to a final tax.
(B) exempt from income taxation.
(C) taxable only if earned by a citizen.
(D) included in the income tax return.
136. Interest income of a domestic commercial bank derived from a peso loan to a domestic corporation in 2016 is:
a) Subject to the 30% income tax based on its net taxable income;
b) Subject to the 20% final withholding tax;
c) Subject to the 7.5% final withholding tax;
d) Subject to 10% final withholding tax.

137. MGC Corp. secured an income tax holiday for 5 years as a pioneer industry. On the fourth year of the tax holiday, MGC Corp.
declared and paid cash dividends to its stockholders, all of whom are individuals. Are the dividends taxable?
(A) The dividends are taxable; the tax exemption of MGC Corp. does not extend to its stockholders.
(B) The dividends are tax exempt because of MGC Corp.’s income tax holiday.
(C) The dividends are taxable if they exceed 50% of MGC Corp.’s retained earnings.
(D) The dividends are exempt if paid before the end of MGC Corp’s fiscal year.

138. ABC Corp. was dissolved and liquidating dividends were declared and paid to the stockholders. What tax consequence follows?
(A) ABC Corp. should deduct a final tax of 10% from the dividends.
(B) The stockholders should declare their gain from their investment and pay income tax at the ordinary rates.
(C) The dividends are exempt from tax.
(D) ABC Corp. should withhold a 10% creditable tax.

139. Which of the following is an exclusion from gross income?


(A) Salaries and wages
(B) Cash dividends
(C) Liquidating dividends after dissolution of a corporation
(D) De minimis benefits
(E) Embezzled money

140. Linus purchased a life annuity for P 500 000.00 which will pay him P 120 000.00 a year. The life expectancy of Linus is 12
years. Under Section 32 of the National Internal Revenue Code of 1997, which of the following will Linus be able to exclude from his
gross income.
a. P 940,000.00
b. P 500,000.00
c. P1,440,000.00
d. P120,000.00

141. Patroclus was injured in a vehicular accident in 2015. He incurred and paid medical expenses of P10,000.00 and legal fees of
P5,000.00 during that year. In 2016, he recovered P35,000.00 as settlement from the insurance company which insured the car
owned by the other party involved in the accident. From the above payments and transactions, the amount taxable to Patroclus in
2016 is:
a) P20,000.00
b) P25,000.00
c) P30,000.00
d) P35,000.00
e) None of the above. All of the recovered amounts are not income because they are merely compensation for actual losses suffered.
They do not constitute taxable gain as they were not received as payment for services, interest or profit from investment.

142. Aleta sued Bobby for breach of promise to marry. Boboy lost the case and duly paid the court's award that included, among
others, P100,000 as moral damages for the mental anguish Aleta suffered. Did Aleta earn a taxable income?
(A) She had a taxable income of P100,000, since income is income from whatever source.
(B) She had no taxable income because it was a donation
(C) She had taxable income since she made a profit.
(D) She had no taxable income since moral damages are compensatory.

LEAD TAX Pre Week Page 28 of 28

You might also like